Although the school would receive financial benefits if it

This topic has expert replies
Legendary Member
Posts: 944
Joined: Wed May 30, 2012 8:21 am
Thanked: 8 times
Followed by:5 members

Timer

00:00

Your Answer

A

B

C

D

E

Global Stats

Although the school would receive financial benefits if it had soft drink vending machines in the cafeteria, we should not allow them. Allowing soft drink machines there would not be in our students' interest. If our students start drinking more soft drinks, they will be less healthy.

The argument depends on which of the following?

(A) If the soft drink vending machines were placed in the cafeteria, students would consume more soft drinks as a result.
(B) The amount of soft drinks that most students at the school currently drink is not detrimental to their health.
(C) Students are apt to be healthier if they do not drink soft drinks at all than if they just drink small amounts occasionally.
(D) Students will not simply bring soft drinks from home if the soft drink vending machines are not placed in the cafeteria.
(E) The school's primary concern should be to promote good health among its students.


OA: A

Source: OG 2016,CR Qs.37

@Verbal Experts - although I got this one correct, took more than 3 mins to nail it. Could you please suggest some faster and smarter way to get this in 2 mins ? Is it a CAUSAL argument ?

Also, please your analysis on why EXACTLY D is wrong ? It seems to be a close choice. Isn't it ?
Last edited by RBBmba@2014 on Thu Jun 23, 2016 5:37 am, edited 1 time in total.

User avatar
Legendary Member
Posts: 2131
Joined: Mon Feb 03, 2014 9:26 am
Location: https://martymurraycoaching.com/
Thanked: 955 times
Followed by:140 members
GMAT Score:800

by MartyMurray » Wed Jun 22, 2016 11:19 pm

Timer

00:00

Your Answer

A

B

C

D

E

Global Stats

RBBmba@2014 wrote:@Verbal Experts - although I got this one correct, took more than 3 mins to nail it. Could you please suggest some faster and smarter way to get this in 2 mins ? Is it a CAUSAL argument ?

Also, please your analysis on why EXACTLY D is wrong ? It seems to be a close choice. Isn't it ?
In my opinion, one way to save time is to not worry about whether the question is a causal argument. Rather, just seek to clearly understand the prompt and to notice its key points.

Soft drink machines should not be allowed ... because allowing them would not be in the students' interest ... because if the students start drinking MORE (key word) soft drinks they will be LESS HEALTHY.

Now read the question. We don't know what the answer will be as we go into the choices, but because we clearly understand the argument, we won't get sucked into a trap.

(A) This answer choice is rather simple, but it totally captures what's necessary for the argument to work. The argument depends on the idea that the students will actually consume more soft drinks if the machines are allowed. I was actually a little surprised by how simple and direct the choice was, and wondered if it were somehow wrong.

(B) The key thing that makes this wrong is LESS HEALTHY, up there in the argument. Whether they are already hurting themselves with soft drinks does not matter. What matters is whether they will be EVEN WORSE OFF if the machines are allowed.

(C) You can cross this one off quickly, because it is so specifically about one little thing, and we don't know whether they already drink large amounts or none or what they do. As there could be many scenarios beyond the one discussed in this answer choice, this statement is just not going to be the right answer.

(D) Trap answer. Yup it sounds right in a way, but the key word, MORE, is the one that separates this from A. The argument does not depend on students' not consuming soft drinks if there are no machines. Rather the argument depends on the students' drinking more soft drinks if there are machines.

(E) Another trap answer. While this may be a valid question, it is beyond the scope of the argument, which is simply about the effects of allowing the machines in the school.

So, simple as it is, the correct answer is A.
Marty Murray
Perfect Scoring Tutor With Over a Decade of Experience
MartyMurrayCoaching.com
Contact me at [email protected] for a free consultation.

Legendary Member
Posts: 944
Joined: Wed May 30, 2012 8:21 am
Thanked: 8 times
Followed by:5 members

by RBBmba@2014 » Thu Jun 23, 2016 10:17 am

Timer

00:00

Your Answer

A

B

C

D

E

Global Stats

Marty - while I get your point (it's a pretty subtle change, I think), though a clarification required.

What is being conveyed through A, doesn't that SAME thing is IMPLICITLY mentioned in the PASSAGE itself - "Allowing soft drink machines [in the cafeteria] there would not be in our students' interest. If our students start drinking more soft drinks, they will be less healthy" ?

So, in that way isn't the OA sort of repetition of what is IMPLICITLY mentioned in the PASSAGE ?

Curious to know your thoughts!

User avatar
Legendary Member
Posts: 2131
Joined: Mon Feb 03, 2014 9:26 am
Location: https://martymurraycoaching.com/
Thanked: 955 times
Followed by:140 members
GMAT Score:800

by MartyMurray » Thu Jun 23, 2016 5:05 pm

Timer

00:00

Your Answer

A

B

C

D

E

Global Stats

RBBmba@2014 wrote:Marty - while I get your point (it's a pretty subtle change, I think), though a clarification required.

What is being conveyed through A, doesn't that SAME thing is IMPLICITLY mentioned in the PASSAGE itself - "Allowing soft drink machines [in the cafeteria] there would not be in our students' interest. If our students start drinking more soft drinks, they will be less healthy" ?

So, in that way isn't the OA sort of repetition of what is IMPLICITLY mentioned in the PASSAGE ?

Curious to know your thoughts!
The point of this question is that there is a gap in the argument.

The argument says the following.

- Allowing soft drink machines in the cafeteria is not in the students' interest.

- If students drink more soft drinks they will be less healthy.

Notice, there is a gap between the first statement and the second, in that nowhere is it made clear that putting the machines in the cafeteria will lead to students drinking more soft drinks.

As you said, that they will drink more is implied, or assumed, but not stated, and the argument DEPENDS on the assumption that if the machines are allowed the students will drink more.

This format, an argument with a gap in, is typical of GMAT CR assumption questions. Generally the correct answer is an assumption that fills the gap.

Maybe you are reacting to this question and the OA similarly to how I did. The correct answer is so obvious and so straightforward that you almost wonder whether you have made a mistake in choosing it.
Marty Murray
Perfect Scoring Tutor With Over a Decade of Experience
MartyMurrayCoaching.com
Contact me at [email protected] for a free consultation.

Legendary Member
Posts: 944
Joined: Wed May 30, 2012 8:21 am
Thanked: 8 times
Followed by:5 members

by RBBmba@2014 » Fri Jun 24, 2016 2:41 am

Timer

00:00

Your Answer

A

B

C

D

E

Global Stats

Marty Murray wrote:Maybe you are reacting to this question and the OA similarly to how I did. The correct answer is so obvious and so straightforward that you almost wonder whether you have made a mistake in choosing it.
Right you're Sir :-)
Marty Murray wrote: As you said, that they will drink more is implied, or assumed, but not stated, and the argument DEPENDS on the assumption that if the machines are allowed the students will drink more.

This format, an argument with a gap in, is typical of GMAT CR assumption questions. Generally the correct answer is an assumption that fills the gap.
I agree with you, but in a few GMAT CR (although can't cite any such offhand ) I've seen that any option is considered incorrect because it's ALREADY IMPLICITLY mentioned (but not stated) in the passage. So, I presume any such CR is COMPARATIVELY rare on GMAT!

Thoughts ?

User avatar
Legendary Member
Posts: 2131
Joined: Mon Feb 03, 2014 9:26 am
Location: https://martymurraycoaching.com/
Thanked: 955 times
Followed by:140 members
GMAT Score:800

by MartyMurray » Fri Jun 24, 2016 5:37 am

Timer

00:00

Your Answer

A

B

C

D

E

Global Stats

RBBmba@2014 wrote:
Marty Murray wrote: As you said, that they will drink more is implied, or assumed, but not stated, and the argument DEPENDS on the assumption that if the machines are allowed the students will drink more.

This format, an argument with a gap in, is typical of GMAT CR assumption questions. Generally the correct answer is an assumption that fills the gap.
I agree with you, but in a few GMAT CR (although can't cite any such offhand ) I've seen that any option is considered incorrect because it's ALREADY IMPLICITLY mentioned (but not stated) in the passage. So, I presume any such CR is COMPARATIVELY rare on GMAT!

Thoughts ?
Let's get something absolutely clear.

While people use all kinds of little rules of thumb to help them to eliminate CR answer choices, among them, too extreme, too specific and restates something said in prompt, there is really only one criterion that matters. Does the answer choice logically answer the question? If it does, then it is the correct answer. If it does not, then it is not.

The fact that something restates something said in the prompt is not what makes the choice wrong. What makes it wrong is that it does not answer the question.

In a weaken question, for instance, something that is already stated in the prompt obviously cannot weaken the argument further by being repeated, but to make things clear, what really matters is that the statement does not weaken the argument, and that's what you have to focus on in choosing between options. Otherwise you end up either worrying about things that don't matter or picking the wrong answer choice because somehow you perceive the right answer choice as breaking some "rule" that is not the point in the first place.

Something similar to this goes on in handling SC questions, with the "changes the meaning from that of the original" concept. The truth is that if the original is not correct, then the correct answer choice may very well convey a meaning different from that of the original. That idea that the correct answer to a SC question cannot have a meaning different from that of the original is another misleading, waste of time, mostly bogus "rule".

So getting right answers to verbal questions consistently and efficiently is best done by using only tight logic rather than gimmicky "rules".

All that having been said, answer choice A to the question being discussed here does not restate something said in the prompt. Nowhere in the prompt is a clear connection made between allowing soft drink machines in the cafeteria and students drinking more soft drinks. Rather, that that connection exists is an assumption upon which the argument so clearly depends that you got the impression that the argument made the connection.

So this question is actually a perfect example of a GMAT assumption question, with a clear gap in the argument that is clearly filled by one of the answer choices. The only thing that maybe makes this question atypical is that the gap and the fact that the answer choice fills the gap are so obvious.
Marty Murray
Perfect Scoring Tutor With Over a Decade of Experience
MartyMurrayCoaching.com
Contact me at [email protected] for a free consultation.

User avatar
Legendary Member
Posts: 2663
Joined: Wed Jan 14, 2015 8:25 am
Location: Boston, MA
Thanked: 1153 times
Followed by:128 members
GMAT Score:770

by DavidG@VeritasPrep » Fri Jun 24, 2016 6:05 am

Timer

00:00

Your Answer

A

B

C

D

E

Global Stats

Marty Murray wrote:
RBBmba@2014 wrote:
Marty Murray wrote: As you said, that they will drink more is implied, or assumed, but not stated, and the argument DEPENDS on the assumption that if the machines are allowed the students will drink more.

This format, an argument with a gap in, is typical of GMAT CR assumption questions. Generally the correct answer is an assumption that fills the gap.
I agree with you, but in a few GMAT CR (although can't cite any such offhand ) I've seen that any option is considered incorrect because it's ALREADY IMPLICITLY mentioned (but not stated) in the passage. So, I presume any such CR is COMPARATIVELY rare on GMAT!

Thoughts ?

Let's get something absolutely clear.

While people use all kinds of little rules of thumb to help them to eliminate CR answer choices, among them, too extreme, too specific and restates something said in prompt, there is really only one criterion that matters. Does the answer choice logically answer the question? If it does, then it is the correct answer. If it does not, then it is not.

The fact that something restates something said in the prompt is not what makes the choice wrong. What makes it wrong is that it does not answer the question.

In a weaken question, for instance, something that is already stated in the prompt obviously cannot weaken the argument further by being repeated, but to make things clear, what really matters is that the statement does not weaken the argument, and that's what you have to focus on in choosing between options. Otherwise you end up either worrying about things that don't matter or picking the wrong answer choice because somehow you perceive the right answer choice as breaking some "rule" that is not the point in the first place.

Something similar to this goes on in handling SC questions, with the "changes the meaning from that of the original" concept. The truth is that if the original is not correct, then the correct answer choice may very well convey a meaning different from that of the original. That idea that the correct answer to a SC question cannot have a meaning different from that of the original is another misleading, waste of time, mostly bogus "rule".

So getting right answers to verbal questions consistently and efficiently is best done by using only tight logic rather than gimmicky "rules".

All that having been said, answer choice A to the question being discussed here does not restate something said in the prompt. Nowhere in the prompt is a clear connection made between allowing soft drink machines in the cafeteria and students drinking more soft drinks. Rather, that that connection exists is an assumption upon which the argument so clearly depends that you got the impression that the argument made the connection.

So this question is actually a perfect example of a GMAT assumption question, with a clear gap in the argument that is clearly filled by one of the answer choices. The only thing that maybe makes this question atypical is that the gap and the fact that the answer choice fills the gap are so obvious.
I could not agree more with Marty's post. One thing I'll add: we're never assuming that the "if" component of an "if/then" construction is true. For example, consider the argument "If Dan could fly, he would be an excellent basketball player." We're obviously not stating that Dan can fly! So an answer choice that revealed that Dan could, in fact, fly would be both new information and quite surprising.
Veritas Prep | GMAT Instructor

Veritas Prep Reviews
Save $100 off any live Veritas Prep GMAT Course

Legendary Member
Posts: 944
Joined: Wed May 30, 2012 8:21 am
Thanked: 8 times
Followed by:5 members

by RBBmba@2014 » Mon Jun 27, 2016 4:26 am

Timer

00:00

Your Answer

A

B

C

D

E

Global Stats

Marty/Dave - Got you guys!
DavidG@VeritasPrep wrote: I could not agree more with Marty's post. One thing I'll add: we're never assuming that the "if" component of an "if/then" construction is true. For example, consider the argument "If Dan could fly, he would be an excellent basketball player." We're obviously not stating that Dan can fly! So an answer choice that revealed that Dan could, in fact, fly would be both new information and quite surprising.
@ Dave - a quick question on your above quote.

I guess, we consider the "if" component (of an "if/then" construction) the PREMISE. Right ?

And can you please shed any light EXACTLY which part is the CONCLUSION of this CR ? (Although I got this CR correct, just curious to know the same!)

User avatar
Legendary Member
Posts: 2663
Joined: Wed Jan 14, 2015 8:25 am
Location: Boston, MA
Thanked: 1153 times
Followed by:128 members
GMAT Score:770

by DavidG@VeritasPrep » Mon Jun 27, 2016 12:17 pm

Timer

00:00

Your Answer

A

B

C

D

E

Global Stats

I guess, we consider the "if" component (of an "if/then" construction) the PREMISE. Right ?
Not necessarily. The entire if/then construction could function as either a premise or a conclusion, depending on how it's used.

As a Premise: If Dave doesn't pick up his daughter once she begins to cry, she will scream all night. Therefore Dave will be very tired tomorrow.

As a Conclusion: Dave's daughter has colic. So if he doesn't pick up her up once she begins to cry, she will scream all night.

Context is everything.
Veritas Prep | GMAT Instructor

Veritas Prep Reviews
Save $100 off any live Veritas Prep GMAT Course

User avatar
Legendary Member
Posts: 2663
Joined: Wed Jan 14, 2015 8:25 am
Location: Boston, MA
Thanked: 1153 times
Followed by:128 members
GMAT Score:770

by DavidG@VeritasPrep » Mon Jun 27, 2016 12:21 pm

Timer

00:00

Your Answer

A

B

C

D

E

Global Stats

And can you please shed any light EXACTLY which part is the CONCLUSION of this CR ? (Although I got this CR correct, just curious to know the same!)
You could make the argument that there's more than one conclusion. The main conclusion seems to be that school should not allow the soft drink vending machines. We then get the intermediate conclusion that the soft drink machines would not be in the students' interest. The if/then construction seems to work as a premise that supports the intermediate conclusion, which leads to the main conclusion. But, as Marty noted, it's more important to understand what the argument is saying than to properly categorize each subcomponent of the argument.
Veritas Prep | GMAT Instructor

Veritas Prep Reviews
Save $100 off any live Veritas Prep GMAT Course

User avatar
Legendary Member
Posts: 698
Joined: Tue Jul 21, 2015 12:12 am
Location: Noida, India
Thanked: 32 times
Followed by:26 members
GMAT Score:740

by richachampion » Mon Oct 03, 2016 3:37 pm

Timer

00:00

Your Answer

A

B

C

D

E

Global Stats

Marty Murray wrote:
Let's get something absolutely clear.

While people use all kinds of little rules of thumb to help them to eliminate CR answer choices, among them, too extreme, too specific and restates something said in prompt, there is really only one criterion that matters. Does the answer choice logically answer the question? If it does, then it is the correct answer. If it does not, then it is not.

The fact that something restates something said in the prompt is not what makes the choice wrong. What makes it wrong is that it does not answer the question.
I like you very much because of your approach and reasoning.


Although the school would receive financial benefits if it had soft drink vending machines in the cafeteria, we should not allow them. - Paradox

Paradox Resolved -Allowing soft drink machines there would not be in our students' interest. If our students start drinking more soft drinks, they will be less healthy.

In short health benefits are overriding the financial benefits.
Therefore,I believe that main discussion point should be between option A and Option B. D is OFS because the question is seeking an assumption that can help us that school doesn't want to be and an additional cause of putting students health at stake.

D is OFS because whether they carry CD or not has no effect on the question "WHETHER OR NOT UNIVERSITY IS PUTTING STUDENTS HEALTH AT STAKE BY THEIR POLICES"

Extra Credit Discussion(may not be relevant for many students) - Although this is not a paradox question, but I was just trying to understand the questions logic and how the question is structured. If this would have been a paradox question then there would be an option stating that -
Allowing soft drink machines there would not be in our students' interest. If our students start drinking more soft drinks, they will be less healthy.

The argument depends on which of the following?

(A) If the soft drink vending machines were placed in the cafeteria, students would consume more soft drinks as a result.
(B) The amount of soft drinks that most students at the school currently drink is not detrimental to their health.

I think that the logical path to eliminate this option is "currently drink" because we are not concerned about what is the current or past situation, but how can university policy effect students Health in future; thats the logic.

Mr. Murray,

Do you think that "most " is for some purpose here and can also(apart from logic that we discussed) be used for eliminating option B
R I C H A,
My GMAT Journey: 470 → 720 → 740
Target Score: 760+
[email protected]
1. Press thanks if you like my solution.
2. Contact me if you are not improving. (No Free Lunch!)

User avatar
Legendary Member
Posts: 2131
Joined: Mon Feb 03, 2014 9:26 am
Location: https://martymurraycoaching.com/
Thanked: 955 times
Followed by:140 members
GMAT Score:800

by MartyMurray » Mon Oct 03, 2016 8:16 pm

Timer

00:00

Your Answer

A

B

C

D

E

Global Stats

richachampion wrote:(B) The amount of soft drinks that most students at the school currently drink is not detrimental to their health.

Mr. Murray,

Do you think that "most " is for some purpose here and can also(apart from logic that we discussed) be used for eliminating option B
Look at the logic. If that answer choice were otherwise correct, the most would not make it incorrect. If allowing soft drink machines in the cafeteria would make most students less healthy, then even though it might not make all of the students less healthy, it would negatively affect the health of students or of the student body in general. So this is not a situation in which one can eliminate an answer choice because of the use of a word like some or most.
Marty Murray
Perfect Scoring Tutor With Over a Decade of Experience
MartyMurrayCoaching.com
Contact me at [email protected] for a free consultation.

User avatar
Legendary Member
Posts: 698
Joined: Tue Jul 21, 2015 12:12 am
Location: Noida, India
Thanked: 32 times
Followed by:26 members
GMAT Score:740

by richachampion » Tue Oct 04, 2016 12:29 am

Timer

00:00

Your Answer

A

B

C

D

E

Global Stats

Marty Murray wrote: Look at the logic. If that answer choice were otherwise correct, the most would not make it incorrect. If allowing soft drink machines in the cafeteria would make most students less healthy, then even though it might not make all of the students less healthy, it would negatively affect the health of students or of the student body in general. So this is not a situation in which one can eliminate an answer choice because of the use of a word like some or most.
OK. THANKS!
R I C H A,
My GMAT Journey: 470 → 720 → 740
Target Score: 760+
[email protected]
1. Press thanks if you like my solution.
2. Contact me if you are not improving. (No Free Lunch!)